Using Parseval's Theorem to evaluate an integral -- Help please

Click For Summary
The discussion focuses on using Parseval's theorem to evaluate the integral ∫ dk sin^4(k)/(k^2) over the range of ±infinity for a piecewise function defined on specific intervals. The user struggles to derive the correct Fourier transform and seeks clarification on the proper formulation of the function f(x) within the given boundaries. There is confusion regarding the integration limits and the adjustment of the exponential factor in the Fourier transform. The user is looking for guidance on starting the Fourier transform equation to proceed with the integration. The conversation highlights the challenges of applying Parseval's theorem in this context.
KeithKp
Messages
4
Reaction score
0

Homework Statement


By applying Parseval's (Plancherel's) theorem to the function
7a9819bffa30e110228e3910e13a9c1.png
are given by:

f(x) = -1 for -2 ≤ x < 0
1 for 0 ≥ x < 2
0 otherwise

determine the value of the following integral.

∫ dk sin^4(k)/(k^2) (Integral between ±infinity)

Homework Equations



Parseval's Theorem, i.e the integral of the modulus squared of a function is equal to the integral of the modulus squared of its Fourier transform.

Fourier Transform formula.

The Attempt at a Solution



I've tried multiple times to try and arrive at the correct answer of pi/2 but I just can't do it. Is the Fourier transform:

∫-dx e^(ikx) + ∫ dx e^(ikx)? (First integral is between -2 and 0, second between 0 and 2). Because I can't get the correct answer doing that.

And for the equation of the function, is it just sgn(x), with the integral between -2 and 2? How else do I write f(x) given those boundaries? If I use sgn(x) between -2 and 2, I get 4 for the left hand side. But I still can't derive that integral.

If someone would just start me off with the correct Fourier transform equation, i'll go away and crunch the integration myself.

Thank you.
 
Last edited by a moderator:
Physics news on Phys.org
KeithKp said:
(First integral is between -2 and 0, second between 0 and 2)
Yes, but then you need to adjust the exponential factor, since eikx is periodic with period 2π, not 4. Try eikπx/2.
 
Question: A clock's minute hand has length 4 and its hour hand has length 3. What is the distance between the tips at the moment when it is increasing most rapidly?(Putnam Exam Question) Answer: Making assumption that both the hands moves at constant angular velocities, the answer is ## \sqrt{7} .## But don't you think this assumption is somewhat doubtful and wrong?

Similar threads

  • · Replies 5 ·
Replies
5
Views
2K
  • · Replies 6 ·
Replies
6
Views
1K
  • · Replies 1 ·
Replies
1
Views
1K
  • · Replies 14 ·
Replies
14
Views
2K
Replies
3
Views
2K
  • · Replies 3 ·
Replies
3
Views
2K
  • · Replies 1 ·
Replies
1
Views
2K
  • · Replies 7 ·
Replies
7
Views
2K
Replies
1
Views
2K
  • · Replies 10 ·
Replies
10
Views
2K